Messaggioda luca.barletta » 21/06/2007, 21:17

la formula è proprio quella. Rileggi come siamo arrivati alla w.
Frivolous Theorem of Arithmetic:
Almost all natural numbers are very, very, very large.
Avatar utente
luca.barletta
Moderatore globale
Moderatore globale
 
Messaggio: 2570 di 4341
Iscritto il: 21/10/2002, 20:09

Messaggioda Giova411 » 21/06/2007, 21:28

$(z-0)/(1-0) = z$ devo leggere anche questo mio POST per caso? Con w al posto di z? Pensavo di aver scritto una merdinetta...

Alla fine Z con le formiche l'ho trovata giusta secondo il tuo immenso sapere? Mio caro PROF! :-D

Così ho le distribuzioni e basta (come Codino mi ha detto e ridetto 15 o 16 volte!) ma in questo caso mi servono solo queste e non le densità. O no? Ma poi chiede di verificare la dipendenza delle nuove W e Z trovate. Che si fa? Mi butto dal balcone? ;-)
Giova411
Advanced Member
Advanced Member
 
Messaggio: 1062 di 2254
Iscritto il: 16/11/2006, 00:34

Messaggioda Giova411 » 21/06/2007, 21:43

Ok, mio grandissimo :prayer: insegnante cercherò di ricostuire il tutto e domani cercherò di dimostrare la loro dip o indip.

Ricordiamoci: VIVA IL CONVOLUTORE! CI SALVERA' + o - TUTTI 8-)
Giova411
Advanced Member
Advanced Member
 
Messaggio: 1063 di 2254
Iscritto il: 16/11/2006, 00:34

Messaggioda Giova411 » 21/06/2007, 22:01

luca.barletta ha scritto:troviamo innanzitutto che $F_X(x)=int_0^x f_X(t)dt=int_0^x 1dt=x$, stessa cosa per Y. Poi nella formula avevamo $F_X(w)=w$ e $F_Y(w)=w$

Capita questa parte! Che distrattone che sono! Capito perché alla fine ottengo il polinomio tutto in $w$.
Ancora devo capire dove, all'inizio, applichiamo la legge uniforme. :oops:
Cioé quale devo prendere? Perché sembra che vadano bene sia questa della densità $(1)/(b-a)$ che quella della distribuzione $(x-a)/(b-a)$. Concettualmente qual é quella da considerare in questo caso? A prima battuta direi quella della distribuzione che mi dà già l'integrale bello e fatto.
Giova411
Advanced Member
Advanced Member
 
Messaggio: 1065 di 2254
Iscritto il: 16/11/2006, 00:34

Messaggioda Giova411 » 22/06/2007, 16:14

Capita tutta la prima parte (GRAZIE). Anche se la partenza non tanto. Perché si parte con $1-F_W(w) = ... $ il resto OK
luca.barletta ha scritto:Prova a ragionare su $1-F_W(w)=1-P[W<w]=P[W>w]=P[min{X,Y}>w]...$


Per verificare che $W$ e $Z$ sono indipendenti devo controllare se:

$F(w,z) = F_1 (w) * F_2 (z)$

$F_1 (w) * F_2 (z)$ le abbiamo trovate (beh :-D , diciamo che le ha trovate Luca)

Come faccio a trovare $F(w,z) $ con i dati che ho? Non ho un densità congiunta sulla quale fare l'integrale doppio così come sono abituato a fare.


L'altro modo di verificare l'eventuale indipendenza, usando le densità:
$f(w,z) = f_1 (w) * f_2 (z)$
Così le densità marginali le posso trovare derivando le distrib trovate. Ma c'é sempre di mezzo la densità congiunta a sx dell'uguaglianza che non so trovare.


Aiutino PLEASE :oops:
Giova411
Advanced Member
Advanced Member
 
Messaggio: 1072 di 2254
Iscritto il: 16/11/2006, 00:34

Messaggioda Piera » 22/06/2007, 19:13

Non ho visto o fatto calcoli.
Secondo me non sono indipendenti.
Infatti ciascuna variabile aleatoria dipende da $x$ e da $y$...
Piera
Average Member
Average Member
 
Messaggio: 758 di 923
Iscritto il: 17/06/2005, 21:43

Messaggioda Giova411 » 22/06/2007, 19:22

Ciao PIE' !!!!!! Quanto tempo!!!!

I calcoli sono giusti perché me li ha spiegati per bene Luca e finalmente li ho capiti.

Solo una cosa: per dimostrare che sono dipendenti devo fare ciò che ho scritto nel Post sopra il tuo (con $!=$ anziché $=$)
Le distr marginali sono: $W=2w-w^2$ e $Z=1-(1-z)^2$.

Ora come mi muovo con queste? Posso trovare le densità marginali derivando. OK. Ma mi serve o la distribuzione congiunta o la densità congiunta. Giusto? Se si, con 2 paroline, mi dici come ci arrivo alle congiunte?
Giova411
Advanced Member
Advanced Member
 
Messaggio: 1075 di 2254
Iscritto il: 16/11/2006, 00:34

Messaggioda Piera » 23/06/2007, 02:54

Scusa per il ritardo ma sono rientrato da poco.
Allora, se fossero indipendenti deve valere l'uguaglianza:
$P(W<=t)P(Z<=a)=P(W<=t,Z<=a)$.
Ora, data che sono quasi sicuro che non lo sono, devi far vedere che ad esempio per $t=a=1/2$ non è verificata l'uguaglianza sopracitata.
Calcola:
$P(W<=1/2)$
$P(Z<=1/2)$
questo è facile.
Per calcolare $P(W<=1/2,Z<=1/2)$ puoi ricorrere alla probabilità condizionata:
$P(min(X,Y)<=1/2,|X-Y|<=1/2)=P(min(X,Y)<=1/2,|X-Y|<=1/2|Y<X)P(Y<X)+P(min(X,Y)<=1/2,|X-Y|<=1/2|Y>X)P(Y>X)$.
Ora, se $Y<X$ allora $min(X,Y)=Y$ e $|X-Y|=X-Y$
e analogamente per $Y>X$.
Pertanto
$P(min(X,Y)<=1/2,|X-Y|<=1/2|Y<X)P(Y<X)+P(min(X,Y)<=1/2,|X-Y|<=1/2|Y>X)P(Y>X)=$
$=P(Y<=1/2,X-Y<=1/2|Y<X)P(Y<X)+P(X<=1/2,Y-X<=1/2|Y>X)P(Y>X)=$
$=(P(Y<=1/2,X-Y<=1/2,Y<X))/(P(Y<X))P(Y<X)+(P(X<=1/2,Y-X<=1/2,Y>X))/(P(Y>X))P(Y>X)=$
$P(Y<=1/2,X-Y<=1/2,Y<X)+P(X<=1/2,Y-X<=1/2,Y>X)$.
Adesso ti rimane da calcolare le ultime due probabilità.
Per la prima devi calcolare all'interno del quadrato di lato unitario l'area delimitata dalle rette (o meglio dai semipiani) $Y<=1/2,X-Y<=1/2,Y<X$. Analogamente per la seconda probabilità.
Ovviamente, se qualcosa non ti è chiaro, chiedi pure.
Piera
Average Member
Average Member
 
Messaggio: 759 di 923
Iscritto il: 17/06/2005, 21:43

Messaggioda Giova411 » 23/06/2007, 09:04

Piera ha scritto:Scusa per il ritardo ma sono rientrato da poco.
Allora, se fossero indipendenti deve valere l'uguaglianza:
$P(W<=t)P(Z<=a)=P(W<=t,Z<=a)$.
Ora, data che sono quasi sicuro che non lo sono, devi far vedere che ad esempio per $t=a=1/2$ non è verificata l'uguaglianza sopracitata.
Calcola:
$P(W<=1/2)$
$P(Z<=1/2)$
questo è facile.

Fin qui OK! Capito pure il discorso generale per dimostrare la loro dipendenza 8-)
Piera ha scritto:Per calcolare $P(W<=1/2,Z<=1/2)$ puoi ricorrere alla probabilità condizionata:
$P(min(X,Y)<=1/2,|X-Y|<=1/2)=P(min(X,Y)<=1/2,|X-Y|<=1/2|Y<X)P(Y<X)+P(min(X,Y)<=1/2,|X-Y|<=1/2|Y>X)P(Y>X)$.
Ora, se $Y<X$ allora $min(X,Y)=Y$ e $|X-Y|=X-Y$
e analogamente per $Y>X$.
Pertanto
$P(min(X,Y)<=1/2,|X-Y|<=1/2|Y<X)P(Y<X)+P(min(X,Y)<=1/2,|X-Y|<=1/2|Y>X)P(Y>X)=$
$=P(Y<=1/2,X-Y<=1/2|Y<X)P(Y<X)+P(X<=1/2,Y-X<=1/2|Y>X)P(Y>X)=$
$=(P(Y<=1/2,X-Y<=1/2,Y<X))/(P(Y<X))P(Y<X)+(P(X<=1/2,Y-X<=1/2,Y>X))/(P(Y>X))P(Y>X)=$
$P(Y<=1/2,X-Y<=1/2,Y<X)+P(X<=1/2,Y-X<=1/2,Y>X)$.
Adesso ti rimane da calcolare le ultime due probabilità.
Per la prima devi calcolare all'interno del quadrato di lato unitario l'area delimitata dalle rette (o meglio dai semipiani) $Y<=1/2,X-Y<=1/2,Y<X$. Analogamente per la seconda probabilità.
Ovviamente, se qualcosa non ti è chiaro, chiedi pure.


Ecco questi passaggi per me sono difficilissimi :oops: ma visti mi sembra di averli digeriti. Ora provo a fare ciò che, pazientemente, mi hai suggerito. :wink:
Giova411
Advanced Member
Advanced Member
 
Messaggio: 1087 di 2254
Iscritto il: 16/11/2006, 00:34

Messaggioda Piera » 23/06/2007, 11:26

Le rette individuano sul quadrato un parallelogramma la cui area dovrebbe essere $1/4$.
Comunque Giova ho notato che in quasi tutti i problemi del tuo prof soltanto l'ultimo punto è un po' più difficile rispetto agli altri, come in questo caso. Se io fossi in te punterei a capire bene tutti i punti tranne l'ultimo. In fin dei conti la probabilità che in un esame esca per due volte un esercizio con un secondo punto simile a questo è pressochè nulla, mentre capire come si trova la distribuzione del minimo, ad esempio, può essere utile perchè può uscire più di una volta all'esame.
Piera
Average Member
Average Member
 
Messaggio: 762 di 923
Iscritto il: 17/06/2005, 21:43

PrecedenteProssimo

Torna a Statistica e probabilità

Chi c’è in linea

Visitano il forum: Nessuno e 1 ospite